Last visit was: 24 Apr 2024, 11:53 It is currently 24 Apr 2024, 11:53

Close
GMAT Club Daily Prep
Thank you for using the timer - this advanced tool can estimate your performance and suggest more practice questions. We have subscribed you to Daily Prep Questions via email.

Customized
for You

we will pick new questions that match your level based on your Timer History

Track
Your Progress

every week, we’ll send you an estimated GMAT score based on your performance

Practice
Pays

we will pick new questions that match your level based on your Timer History
Not interested in getting valuable practice questions and articles delivered to your email? No problem, unsubscribe here.
Close
Request Expert Reply
Confirm Cancel
SORT BY:
Date
Tags:
Difficulty: 555-605 Levelx   Complete the Passagex                  
Show Tags
Hide Tags
User avatar
Manhattan Prep Instructor
Joined: 30 Apr 2012
Posts: 782
Own Kudos [?]: 2583 [0]
Given Kudos: 5
Send PM
avatar
Manager
Manager
Joined: 07 Dec 2009
Posts: 69
Own Kudos [?]: 102 [0]
Given Kudos: 375
GMAT Date: 12-03-2014
Send PM
User avatar
Manhattan Prep Instructor
Joined: 30 Apr 2012
Posts: 782
Own Kudos [?]: 2583 [1]
Given Kudos: 5
Send PM
Intern
Intern
Joined: 30 Jan 2018
Posts: 12
Own Kudos [?]: 10 [0]
Given Kudos: 153
Location: India
GMAT 1: 660 Q50 V29
GMAT 2: 740 Q49 V41
GPA: 3.17
Send PM
Re: A recent government study links the high rates of respiratory ailments [#permalink]
betterscore wrote:
Which of the following most logically completes the passage?

A recent government study links the high rates of respiratory ailments in Groverston to airborne pollutants released by the Woodco plywood manufacturing plant there. To address the problem, the government imposed strict regulations on emissions which will go into effect in four years. Although Woodco plans to cut its emissions in half two years ahead of schedule, it is unlikely that the
rate of respiratory ailments will decline before the regulations go into effect, since _____________ .

(A) the number of facilities capable of treating respiratory ailments is not likely to increase

(B) reducing emissions even further than planned would necessitate decreasing production at Woodco

(C) it is difficult to make accurate, long-term predictions about emissions

(D) not all respiratory ailments are caused by airborne pollutants

(E) three new plywood manufacturing plants are about to go into production in Groverston





I have a question regarding the wording used in the questions above. For option E to be the valid answer, shouldn't the phase "before the regulations go into effect" not be included in the question as if E is the OA then even after the regulations are put into effect there will not be a decrease in the rates of respiratory ailments.
GMAT Club Verbal Expert
Joined: 13 Aug 2009
Status: GMAT/GRE/LSAT tutors
Posts: 6919
Own Kudos [?]: 63657 [1]
Given Kudos: 1773
Location: United States (CO)
GMAT 1: 780 Q51 V46
GMAT 2: 800 Q51 V51
GRE 1: Q170 V170

GRE 2: Q170 V170
Send PM
Re: A recent government study links the high rates of respiratory ailments [#permalink]
1
Kudos
Expert Reply
abhigulia3006 wrote:
betterscore wrote:
Which of the following most logically completes the passage?

A recent government study links the high rates of respiratory ailments in Groverston to airborne pollutants released by the Woodco plywood manufacturing plant there. To address the problem, the government imposed strict regulations on emissions which will go into effect in four years. Although Woodco plans to cut its emissions in half two years ahead of schedule, it is unlikely that the
rate of respiratory ailments will decline before the regulations go into effect, since _____________ .

(A) the number of facilities capable of treating respiratory ailments is not likely to increase

(B) reducing emissions even further than planned would necessitate decreasing production at Woodco

(C) it is difficult to make accurate, long-term predictions about emissions

(D) not all respiratory ailments are caused by airborne pollutants

(E) three new plywood manufacturing plants are about to go into production in Groverston





I have a question regarding the wording used in the questions above. For option E to be the valid answer, shouldn't the phase "before the regulations go into effect" not be included in the question as if E is the OA then even after the regulations are put into effect there will not be a decrease in the rates of respiratory ailments.

Why can't the rates of respiratory ailments go down after the regulations are put into effect? Sure, there will be more plants, but if they are all subject to "strict regulations on emissions", then the combined emissions of the three plants could be less than current emissions of Woodco.

Regardless, we need to explain why "it is unlikely that the rate of respiratory ailments will decline before the regulations go into effect." We don't care about what happens after the regulations are put into effect. Woodco is cutting its emissions in half, but there will soon be three new plants. BEFORE the regulations go into effect, those new plants can emit as much pollution as they want. So, even though Woodco is going to reduce its emissions, it is unlikely that the total amount of airborne pollution will decrease until the regulations go into effect.

I hope that helps!
Manager
Manager
Joined: 24 Mar 2018
Posts: 198
Own Kudos [?]: 41 [0]
Given Kudos: 288
Location: India
GMAT 1: 680 Q50 V31
Send PM
Re: A recent government study links the high rates of respiratory ailments [#permalink]
KyleWiddison wrote:
To answer this question, you need to find an answer choice that correctly describes why ailments will not decrease in spite of emmission reductions. It's okay to bring in new information, but that information needs to stay true to the stated premises in the argument.

A-We're concerned about the number of ailments, not the treatment "post-ailment".
B-How Woodco acheives its reduction in emmissions is irrelevant because that reduction is already a given premise.
C-Irrelevant information. This does not explain why ailments would increase despite decreased emmissions.
D-Interesting point, but irrelevant because the first premise states that the ailments are connected to airborne emmissions. Granted there could be other ailments, but the ones we care about result from airborne pollutants.
E-This is right on! If new plants go into production in Groverston, the emmissions from the 3 new plants will overwhelm the emmissions acheived in the Woodco plant.

KW


As per my understanding new info is not allowed in such question. Can you please clearify ?
GMAT Club Verbal Expert
Joined: 13 Aug 2009
Status: GMAT/GRE/LSAT tutors
Posts: 6919
Own Kudos [?]: 63657 [0]
Given Kudos: 1773
Location: United States (CO)
GMAT 1: 780 Q51 V46
GMAT 2: 800 Q51 V51
GRE 1: Q170 V170

GRE 2: Q170 V170
Send PM
Re: A recent government study links the high rates of respiratory ailments [#permalink]
Expert Reply
teaserbae wrote:
KyleWiddison wrote:
To answer this question, you need to find an answer choice that correctly describes why ailments will not decrease in spite of emmission reductions. It's okay to bring in new information, but that information needs to stay true to the stated premises in the argument.

A-We're concerned about the number of ailments, not the treatment "post-ailment".
B-How Woodco acheives its reduction in emmissions is irrelevant because that reduction is already a given premise.
C-Irrelevant information. This does not explain why ailments would increase despite decreased emmissions.
D-Interesting point, but irrelevant because the first premise states that the ailments are connected to airborne emmissions. Granted there could be other ailments, but the ones we care about result from airborne pollutants.
E-This is right on! If new plants go into production in Groverston, the emmissions from the 3 new plants will overwhelm the emmissions acheived in the Woodco plant.

KW


As per my understanding new info is not allowed in such question. Can you please clearify ?

As a test-taker, it's generally a terrible idea for you to bring in your own outside knowledge when you're answering GMAT CR questions. But depending on how the question is constructed, the correct answer might be something that did not actually appear in the passage.

For example, think about an assumption question. The correct answer will always be an assumption that the author made in the passage -- so by definition, the correct answer is NOT something that was stated in the passage.

The same is true of this passage, and I think that's what Kyle was saying in his explanation. Check out that last line of the passage again:

    "...it is unlikely that the rate of respiratory ailments will decline before the regulations go into effect, since _____________ ."

The correct answer will strengthen the idea that "it is unlikely that respiratory ailments will decline before regulations go into effect." And of course, that "strengthener" will be an additional piece of information that was not already mentioned in the passage.

I hope that helps!
Manager
Manager
Joined: 21 Jan 2018
Status:Improving everyday
Posts: 109
Own Kudos [?]: 67 [0]
Given Kudos: 64
Location: India
GMAT 1: 660 Q49 V31
GPA: 3.23
WE:Engineering (Aerospace and Defense)
Send PM
Re: A recent government study links the high rates of respiratory ailments [#permalink]
KyleWiddison wrote:
To answer this question, you need to find an answer choice that correctly describes why ailments will not decrease in spite of emmission reductions. It's okay to bring in new information, but that information needs to stay true to the stated premises in the argument.

A-We're concerned about the number of ailments, not the treatment "post-ailment".
B-How Woodco acheives its reduction in emmissions is irrelevant because that reduction is already a given premise.
C-Irrelevant information. This does not explain why ailments would increase despite decreased emmissions.
D-Interesting point, but irrelevant because the first premise states that the ailments are connected to airborne emmissions. Granted there could be other ailments, but the ones we care about result from airborne pollutants.
E-This is right on! If new plants go into production in Groverston, the emmissions from the 3 new plants will overwhelm the emmissions acheived in the Woodco plant.

KW


I have one question
:)

It is regarding option E:It is not necessary that three new plants will emit the same harmful pollution
if we consider this point then option D is better.
Please help

thanks in advance
GMAT Club Verbal Expert
Joined: 13 Aug 2009
Status: GMAT/GRE/LSAT tutors
Posts: 6919
Own Kudos [?]: 63657 [2]
Given Kudos: 1773
Location: United States (CO)
GMAT 1: 780 Q51 V46
GMAT 2: 800 Q51 V51
GRE 1: Q170 V170

GRE 2: Q170 V170
Send PM
Re: A recent government study links the high rates of respiratory ailments [#permalink]
1
Kudos
1
Bookmarks
Expert Reply
ash124 wrote:
KyleWiddison wrote:
To answer this question, you need to find an answer choice that correctly describes why ailments will not decrease in spite of emmission reductions. It's okay to bring in new information, but that information needs to stay true to the stated premises in the argument.

A-We're concerned about the number of ailments, not the treatment "post-ailment".
B-How Woodco acheives its reduction in emmissions is irrelevant because that reduction is already a given premise.
C-Irrelevant information. This does not explain why ailments would increase despite decreased emmissions.
D-Interesting point, but irrelevant because the first premise states that the ailments are connected to airborne emmissions. Granted there could be other ailments, but the ones we care about result from airborne pollutants.
E-This is right on! If new plants go into production in Groverston, the emmissions from the 3 new plants will overwhelm the emmissions acheived in the Woodco plant.

KW


I have one question
:)

It is regarding option E:It is not necessary that three new plants will emit the same harmful pollution
if we consider this point then option D is better.
Please help

thanks in advance

From the passage, we know that Woodco plans to reduce its respiratory-ailment-inducing pollution in two years, but for some reason the rate of respiratory ailments will not decline for four years. The correct answer will explain why the rate of respiratory ailments won't decline sooner. With that in mind, take another look at (D):

Quote:
(D) not all respiratory ailments are caused by airborne pollutants

(D) does not explain why the rate of ailments will not decrease as Woodco decreases pollution. The respiratory ailments caused by factors other than pollution will occur regardless of Woodco's efforts, and we have no information that implies that the rate these other ailments will change in the next several years. So, the overall number of ailments would look something like this:

Before Woodco reduces pollution:
  • Some number of ailments NOT caused by pollution +
  • Some number of ailments caused by pollution

After Woodco reduces pollution in two years:
  • The SAME number of ailments NOT caused by pollution +
  • A REDUCED number of ailments caused by pollution

As you can see, even given the information in (D) we would expect the overall rate of respiratory ailments to decrease in two years. Because (D) does not explain why this reduction in respiratory ailments will not occur, we can eliminate (D).

(E), on the other hand, provides a perfectly reasonable explanation as to why the rate of ailments will not go down as Woodco reduces pollution -- three more plywood plants are about to go into production. It is true that we do not know whether these plants produce the same pollutants, but the strong possibility that they do provides the "most logical" completion to the passage.

I hope that helps!
Senior Manager
Senior Manager
Joined: 20 Dec 2020
Posts: 287
Own Kudos [?]: 30 [0]
Given Kudos: 496
Location: India
Send PM
Re: A recent government study links the high rates of respiratory ailments [#permalink]
KarishmaB GMATNinja

it is unlikely that the rate of respiratory ailments will decline before the regulations go into effect, since _________

(D) not all respiratory ailments are caused by airborne pollutants

If D is true, there are other reasons exist for respiratory ailments, so other factor would lead to increase in ailments. This could be the reason that rate of ailments unlikely to decrease.
Why D is incorrect?
GMAT Club Verbal Expert
Joined: 13 Aug 2009
Status: GMAT/GRE/LSAT tutors
Posts: 6919
Own Kudos [?]: 63657 [0]
Given Kudos: 1773
Location: United States (CO)
GMAT 1: 780 Q51 V46
GMAT 2: 800 Q51 V51
GRE 1: Q170 V170

GRE 2: Q170 V170
Send PM
Re: A recent government study links the high rates of respiratory ailments [#permalink]
Expert Reply
Sneha2021 wrote:
KarishmaB GMATNinja

it is unlikely that the rate of respiratory ailments will decline before the regulations go into effect, since _________

(D) not all respiratory ailments are caused by airborne pollutants

If D is true, there are other reasons exist for respiratory ailments, so other factor would lead to increase in ailments. This could be the reason that rate of ailments unlikely to decrease.
Why D is incorrect?

The conclusion states that it is unlikely that the rate of respiratory ailments will decline. It doesn't say that the ailments are unlikely to be eliminated.

The question then asks us to complete the passage. In other words, why is it unlikely that the rate of respiratory ailments will decline? With that in mind, here's (D):

Quote:
(D) not all respiratory ailments are caused by airborne pollutants

It's important to pay close attention to the precise wording of the answer choice here. (D) states that "not ALL respiratory ailments are caused by airborne pollutants." In other words, SOME (maybe 1%, maybe 99%) respiratory ailments are caused by things other than airborne pollutants.

The problem is that we still have reason to believe that OTHER (maybe 1%, maybe 99%) respiratory ailments ARE caused by airborne pollutants. So, Woodco's plans to cut emissions will chip away at the cause of some cases of respiratory ailments (not ALL, but some). But even chipping away at SOME of the cases of respiratory ailments will still decrease the overall rate. So, we don't have reason to believe that a decrease in the rate is unlikely, and we can eliminate (D).

I hope that helps!
Intern
Intern
Joined: 27 Jun 2020
Posts: 24
Own Kudos [?]: 2 [0]
Given Kudos: 38
Send PM
Re: A recent government study links the high rates of respiratory ailments [#permalink]
Dear Sir GMATNinja
is option E equivalent to “there are other factors which are increasing polution” ?

GMATNinja wrote:
Sneha2021 wrote:
KarishmaB GMATNinja

it is unlikely that the rate of respiratory ailments will decline before the regulations go into effect, since _________

(D) not all respiratory ailments are caused by airborne pollutants

If D is true, there are other reasons exist for respiratory ailments, so other factor would lead to increase in ailments. This could be the reason that rate of ailments unlikely to decrease.
Why D is incorrect?

The conclusion states that it is unlikely that the rate of respiratory ailments will decline. It doesn't say that the ailments are unlikely to be eliminated.

The question then asks us to complete the passage. In other words, why is it unlikely that the rate of respiratory ailments will decline? With that in mind, here's (D):

Quote:
(D) not all respiratory ailments are caused by airborne pollutants

It's important to pay close attention to the precise wording of the answer choice here. (D) states that "not ALL respiratory ailments are caused by airborne pollutants." In other words, SOME (maybe 1%, maybe 99%) respiratory ailments are caused by things other than airborne pollutants.

The problem is that we still have reason to believe that OTHER (maybe 1%, maybe 99%) respiratory ailments ARE caused by airborne pollutants. So, Woodco's plans to cut emissions will chip away at the cause of some cases of respiratory ailments (not ALL, but some). But even chipping away at SOME of the cases of respiratory ailments will still decrease the overall rate. So, we don't have reason to believe that a decrease in the rate is unlikely, and we can eliminate (D).

I hope that helps!


Posted from my mobile device
GMAT Club Verbal Expert
Joined: 13 Aug 2009
Status: GMAT/GRE/LSAT tutors
Posts: 6919
Own Kudos [?]: 63657 [2]
Given Kudos: 1773
Location: United States (CO)
GMAT 1: 780 Q51 V46
GMAT 2: 800 Q51 V51
GRE 1: Q170 V170

GRE 2: Q170 V170
Send PM
Re: A recent government study links the high rates of respiratory ailments [#permalink]
2
Kudos
Expert Reply
TrungTiger wrote:
Dear Sir GMATNinja

is option E equivalent to “there are other factors which are increasing polution” ?

Yes, (E) tells us that pollution is likely to increase in the near future.

Specifically, three new plywood manufacturing plants are about to go into production in Groverston. Because we know that the existing plywood manufacturing plant causes air pollution, it is likely that these additional plants will also cause air pollution. Additionally, we know that the particular type of air pollution released by the current plant is linked to respiratory illness, so it's likely that the pollution from the additional plants will also be linked to respiratory illness.

Finally, because these plants are "about" to go into production, they have the next four years to produce as much pollution as they want before the new regulations come into effect.

All of this gives us reason to believe that the rate of respiratory illness is unlikely to decline before the regulations are enforced. (E) is the correct answer.

I hope that helps!
Intern
Intern
Joined: 27 Jun 2020
Posts: 24
Own Kudos [?]: 2 [1]
Given Kudos: 38
Send PM
Re: A recent government study links the high rates of respiratory ailments [#permalink]
1
Kudos
Thank you very much for your detail explanation.
I comprehended it.

GMATNinja wrote:
TrungTiger wrote:
Dear Sir GMATNinja

is option E equivalent to “there are other factors which are increasing polution” ?

Yes, (E) tells us that pollution is likely to increase in the near future.

Specifically, three new plywood manufacturing plants are about to go into production in Groverston. Because we know that the existing plywood manufacturing plant causes air pollution, it is likely that these additional plants will also cause air pollution. Additionally, we know that the particular type of air pollution released by the current plant is linked to respiratory illness, so it's likely that the pollution from the additional plants will also be linked to respiratory illness.

Finally, because these plants are "about" to go into production, they have the next four years to produce as much pollution as they want before the new regulations come into effect.

All of this gives us reason to believe that the rate of respiratory illness is unlikely to decline before the regulations are enforced. (E) is the correct answer.

I hope that helps!
Senior Manager
Senior Manager
Joined: 23 Dec 2022
Posts: 318
Own Kudos [?]: 35 [0]
Given Kudos: 199
Send PM
Re: A recent government study links the high rates of respiratory ailments [#permalink]
The passage discusses a government study linking the high rates of respiratory ailments in Groverston to airborne pollutants released by the Woodco plywood manufacturing plant. The government has imposed strict regulations on emissions that will go into effect in four years. Although Woodco plans to cut its emissions in half two years ahead of schedule, the passage states that it is unlikely that the rate of respiratory ailments will decline before the regulations go into effect. The question asks us to select the option that most logically completes the passage based on the information provided.

To find the most suitable completion for the passage, we need to consider the reasoning provided and identify the option that provides a logical explanation for why the rate of respiratory ailments is unlikely to decline before the regulations go into effect.

Option (A) states that the number of facilities capable of treating respiratory ailments is not likely to increase. While this information may be relevant to the overall healthcare system, it does not directly address why the rate of respiratory ailments is unlikely to decline before the regulations go into effect. Therefore, option (A) is not the most appropriate completion for the passage.

Option (B) suggests that reducing emissions even further than planned would require decreasing production at Woodco. While this may be a valid point, it does not directly explain why the rate of respiratory ailments is unlikely to decline before the regulations go into effect. Therefore, option (B) is not the most suitable completion for the passage.

Option (C) states that it is difficult to make accurate, long-term predictions about emissions. While this may be true, it does not directly address why the rate of respiratory ailments is unlikely to decline before the regulations go into effect. Hence, option (C) is not the most logical completion for the passage.

Option (D) mentions that not all respiratory ailments are caused by airborne pollutants. This option provides an alternative explanation for the high rates of respiratory ailments, suggesting that there may be other factors contributing to the problem besides Woodco's emissions. Option (D) logically completes the passage by presenting an alternative explanation for the lack of expected decline in respiratory ailments.

Option (E) states that three new plywood manufacturing plants are about to go into production in Groverston. This information suggests that the overall emissions in the area may increase due to the additional plants, which could counteract the emissions reduction efforts by Woodco. Option (E) provides a logical explanation for why the rate of respiratory ailments is unlikely to decline before the regulations go into effect, as the presence of new plants could offset the emissions reductions by Woodco.

In conclusion, both option (D) and option (E) provide plausible completions for the passage. However, option (E) directly addresses the issue of emissions from other plants potentially hindering the decline in respiratory ailments, aligning more closely with the information provided. Therefore, option (E) is the most suitable completion for the passage.
GMAT Club Bot
Re: A recent government study links the high rates of respiratory ailments [#permalink]
   1   2 
Moderators:
GMAT Club Verbal Expert
6919 posts
GMAT Club Verbal Expert
238 posts
CR Forum Moderator
832 posts

Powered by phpBB © phpBB Group | Emoji artwork provided by EmojiOne